- #1
chartery
- 40
- 4
I'm having trouble(s) showing that unit polar bases do not commute.
Adapting <https://math.stackexchange.com/questions/3288981>
taking: ##\hat{\theta} = \frac{1}{r}\frac{\partial }{\partial \theta} ( =\frac{1}{r}\overrightarrow{e}_{\theta})##
then ##\hat{r}\hat{\theta} = \frac{\partial }{\partial r}\left( \frac{1}{r} \frac{\partial f}{\partial \theta}\right)## by chain rule will differ from ##\hat{\theta}\hat{r} = \frac{1}{r}\frac{\partial }{\partial \theta}\left( \frac{\partial f}{\partial r}\right)##
so far so good, but...in <https://physics.stackexchange.com/questions/198280> (first answer)
(via ##\frac{\partial ^{2}x}{\partial \hat{r}\partial \hat{\theta}} vs \frac{\partial ^{2}x}{\partial \hat{\theta}\partial \hat{r}}##)
##\overrightarrow{e_{\hat{r}}}=\overrightarrow{e_{r}}## seems to justify ##(\frac{\partial }{\partial \hat{r}}\left( -sin\theta \right)= )##, ##\frac{\partial }{\partial \hat{r}}\left( -\frac{y}{r} \right)=\frac{\partial }{\partial r}\left( -\frac{y}{r} \right)##
but I don't see how to apply ##\frac{\partial }{\partial \hat{\theta}}## in ##\frac{\partial }{\partial \hat{\theta}}\left( cos\theta \right)##Finally, when just multiplying out the commutator of Cartesian versions, everything seems to cancel:
##[\hat{r},\hat{\theta}] = \left( cos\theta\overrightarrow{x}+sin\theta\overrightarrow{y} \right)\left( -sin\theta\overrightarrow{x}+cos\theta\overrightarrow{y} \right)-\left( -sin\theta\overrightarrow{x}+cos\theta\overrightarrow{y} \right)\left( cos\theta\overrightarrow{x}+sin\theta\overrightarrow{y} \right)## =0??Evidently, my grasp of the underlying logic of the various manipulations is flimsy. As Jim Hacker said, can you tell me what I don't know :-)?
Adapting <https://math.stackexchange.com/questions/3288981>
taking: ##\hat{\theta} = \frac{1}{r}\frac{\partial }{\partial \theta} ( =\frac{1}{r}\overrightarrow{e}_{\theta})##
then ##\hat{r}\hat{\theta} = \frac{\partial }{\partial r}\left( \frac{1}{r} \frac{\partial f}{\partial \theta}\right)## by chain rule will differ from ##\hat{\theta}\hat{r} = \frac{1}{r}\frac{\partial }{\partial \theta}\left( \frac{\partial f}{\partial r}\right)##
so far so good, but...in <https://physics.stackexchange.com/questions/198280> (first answer)
(via ##\frac{\partial ^{2}x}{\partial \hat{r}\partial \hat{\theta}} vs \frac{\partial ^{2}x}{\partial \hat{\theta}\partial \hat{r}}##)
##\overrightarrow{e_{\hat{r}}}=\overrightarrow{e_{r}}## seems to justify ##(\frac{\partial }{\partial \hat{r}}\left( -sin\theta \right)= )##, ##\frac{\partial }{\partial \hat{r}}\left( -\frac{y}{r} \right)=\frac{\partial }{\partial r}\left( -\frac{y}{r} \right)##
but I don't see how to apply ##\frac{\partial }{\partial \hat{\theta}}## in ##\frac{\partial }{\partial \hat{\theta}}\left( cos\theta \right)##Finally, when just multiplying out the commutator of Cartesian versions, everything seems to cancel:
##[\hat{r},\hat{\theta}] = \left( cos\theta\overrightarrow{x}+sin\theta\overrightarrow{y} \right)\left( -sin\theta\overrightarrow{x}+cos\theta\overrightarrow{y} \right)-\left( -sin\theta\overrightarrow{x}+cos\theta\overrightarrow{y} \right)\left( cos\theta\overrightarrow{x}+sin\theta\overrightarrow{y} \right)## =0??Evidently, my grasp of the underlying logic of the various manipulations is flimsy. As Jim Hacker said, can you tell me what I don't know :-)?